Đến nội dung

Hình ảnh

$\boxed{\text{Chuyên Đề}}$ Hình học


  • Please log in to reply
Chủ đề này có 61 trả lời

#1
Viet Hoang 99

Viet Hoang 99

    $\textbf{Trương Việt Hoàng}$

  • Điều hành viên THPT
  • 2291 Bài viết

Đây là file ôn thi hình học cho các bạn thi vào THPT:
File gửi kèm  Hinhhoc.pdf   447.88K   11564 Số lần tải

 

580951_262240127213543_2124880669_n.jpg

 

 

Còn bây giờ chúng ta cùng học để ôn thi HSG hoặc chuyên

 

Trước hết là các phần mềm để vẽ hình.

 

Bản GSP full
GSP viet ( tiếng Việt ) : nên có cái này cho tiện.
Bản hướng dẫn sử dụng.

Xem thêm tại đây

Cài đặt GSP mã là thongnong VUAUJR

mật khẩu D7F674FA

(Nếu không cài mật mã thì vẫn có thể dùng thử được)

 

 

-Lưu ý: Sau khi vẽ xong hình bằng phần mềm $GSP$ thì ta ấn nút $\boxed{Print Screen}$ ở bên phải nút $\boxed{F12}$ sau đó vào phần mềm $Paint$ và ấn tổ hợp phím $CTRL+V$, sau đó lưu hình lại dưới dạng file .PNG (Nếu đã cài mật mã thì có thể ấn CTRL+A rồi ấn CTRL+C rồi vào Paint ấn CTRL+V)

untitled.PNG

Sau đó vào bạn trả lời ở TOPIC thì chọn 

untitled1.PNG

Sau đó chọn 

untitled2.PNG

Sau đó chọn hình của bài giải và ấn $Open$

 

P/s: Đừng dùng $URL$ ảnh vì có thể sau này ảnh sẽ bị lỗi không nhìn thấy được, hãy làm như hướng dẫn bên trên

________________________________

 

 

Phần này để ôn thi HSG lớp 9 môn Hình Học

 

 

 

Chuyên đề Hình Học

Định lý Ta-let - Tam giác đồng dạng (2 cái này thì không cần phải nói nữa nhé)
 

$I-$ Định lý $Ce-va$:

 

Bài toán:

Cho tam giác $ABC$, $D;E;F$ thuộc đường thẳng $BC;CA;AB$ 

Cmr: $AD;BE;CF$ đồng quy $\Leftrightarrow \frac{AF}{FB}.\frac{BD}{DC}.\frac{CE}{EA}=1$

Lời giải:

h.PNG

 

  • Chứng minh phần thuận:

 

Nếu có:  $AD;BE;CF$ đồng quy

thì ta sẽ chứng minh: $\frac{AF}{FB}.\frac{BD}{DC}.\frac{CE}{EA}=1$

 

Giả sử $AD;BE;CF$ đồng quy tại $O$ thì $O$ ở trong hay ở ngoài tam giác $ABC$ ta đều có:
$\frac{AF}{FB}=\frac{S_{AFO}}{S_{BFO}}=\frac{S_{CAF}}{S_{CBF}}=\frac{S_{CAF}-S_{AFO}}{S_{CBF}-S_{BFO}}=\frac{S_{AOC}}{S_{BOC}}$

Chứng minh tương tự (Cmtt) :
+) $\frac{BD}{CD}=\frac{S_{AOB}}{S_{AOC}}$

+) $\frac{CE}{EA}=\frac{S_{BOC}}{S_{AOB}}=1$

Nhân 3 đẳng thức trên lại ta được:

$\frac{AF}{FB}.\frac{BD}{DC}.\frac{CE}{EA}=1$

  • Chứng minh phần đảo:

Nếu có: $\frac{AF}{FB}.\frac{BD}{DC}.\frac{CE}{EA}=1$

thì ta sẽ chứng minh: $AD;BE;CF$ đồng quy

Gọi $BE\cap CF\equiv O$; $AO\cap BC\equiv D'$

Ta có: $\left\{\begin{matrix}\frac{AF}{FB}.\frac{BD}{DC}.\frac{CE}{EA}=1  &  & \\ \frac{AF}{BF}.\frac{BD'}{CD'}.\frac{CE}{AE}=1 (*) &  &  \end{matrix}\right.$

(Chứng minh $(*)$ giống phần thuận)

$\Rightarrow \frac{BD}{CD}=\frac{BD'}{CD'}\Rightarrow \frac{BD}{DC\pm BD}=\frac{BD'}{CD'\pm BD'}$ (Do có 2 truờng hợp)
$\Rightarrow \frac{BD}{BC}=\frac{BD'}{BC}\Rightarrow BD=BD'$
Cmtt $\Rightarrow DC=D'C$

Từ 2 điều trên $\Rightarrow D\equiv D'$

Vậy $AD;BE;CF$ đồng quy.

 

$II-$ Định lý $Menelauyt$ ($Menelaus$)

 

Bài toán:

Cho tam giác $ABC$. $D;E;F$ thuộc đường thẳng $BC;CA;AB$.

Cmr: $D;E;F$ thẳng hàng $\Leftrightarrow \frac{AF}{BF}.\frac{BD}{CD}.\frac{CE}{AE}=1$
Lời giải:

h.PNG

 

  • Chứng minh phần thuận

Nếu có $D;E;F$ thẳng hàng

thì ta sẽ chứng minh: $\frac{AF}{BF}.\frac{BD}{CD}.\frac{CE}{AE}=1$

Giả sử $D;E;F$ thẳng hàng.

Kẻ $AQ//BC$ ($Q\in DF$)

Theo định lý Ta-let ta có:

+) $\frac{AF}{BF}=\frac{QF}{FD}=\frac{AQ}{BD}$

+) $\frac{CE}{EA}=\frac{DC}{AQ}$

Nhân 2 đẳng thức trên ta có:

$\frac{AF}{BF}.\frac{CE}{EA}=\frac{AQ}{BD}.\frac{DC}{AQ}=\frac{DC}{BD}$

$\Leftrightarrow \frac{AF}{BF}.\frac{BD}{CD}.\frac{CE}{AE}=\frac{BD}{CD}.\frac{CD}{BD}=1$

  • Chứng minh phần đảo

 

Nếu có $\frac{AF}{BF}.\frac{BD}{CD}.\frac{CE}{AE}=1$

thì ta sẽ chứng minh: $D;E;F$ thẳng hàng

Giả sử ta có: $\frac{AF}{BF}.\frac{BD}{CD}.\frac{CE}{AE}=1$

Gọi $FD\cap AC\equiv E'$

Ta có: $\left\{\begin{matrix}\frac{AF}{BF}.\frac{BD}{CD}.\frac{CE}{AE}=1  &  & \\ \frac{AF}{BF}.\frac{BD}{CD}.\frac{CE'}{AE'}=1 (*)  &  &  \end{matrix}\right.$

 

 

 

(Chứng minh $(*)$ giống phần thuận)

$\Rightarrow \frac{CE}{AE}=\frac{CE'}{AE'}\Rightarrow \frac{CE}{AE+CE}=\frac{CE'}{AE'+CE'}\Rightarrow \frac{CE}{AC}=\frac{CE'}{AC}\Rightarrow CE=CE'$

Cmtt $\Rightarrow BE=BE'$

$\Rightarrow E\equiv E'$

Vậy $D:E;F$ thẳng hàng

 

-------------------

Tiếp theo mình sẽ đăng các bài tập, bài làm rồi sẽ được tô màu đỏ.


Bài viết đã được chỉnh sửa nội dung bởi Viet Hoang 99: 27-07-2014 - 07:26


#2
Viet Hoang 99

Viet Hoang 99

    $\textbf{Trương Việt Hoàng}$

  • Điều hành viên THPT
  • 2291 Bài viết

Bài tập:

$1/$ Cho hình thang $ABCD$ có $BC//AD$; $BC<AD$, kéo dài $AC$ về phía $C$ và lấy $P$ tùy ý. Gọi $K;L$ là trung điểm $BC;AD$. $PK\cap AB\equiv M$; $PL\cap CD\equiv N$

Cmr: $MN//BC$

 

$2/$ Cho tam giác $ABC$ có $AB>AC$. Qua $C$ kẻ đường thẳng vuông góc với phân giác $BE$ của $\widehat{ABC}$ tại $F$, cắt trung tuyến $BD$ tại $G$. 

Cmr: $DF$ đi qua trung điểm của $GE$

 

$3/$ Cho tam giác $ABC$, trọng tâm $G$. Đường thẳng đi qua $G$ cắt $AB;AC$ tại $M;N$. Xác định vị trí của $M;N$ sao cho: $P=\frac{AB.AC}{AM.AN}$ đạt giá trị lớn nhất

 

$4/$ Cho tam giác $ABC$ cạnh $a;b;c$. $M\in AB; N\in AC$

Cmr:

a) $S_{ABC}=\frac{1}{2}bcsinA=\frac{1}{2}absinC=\frac{1}{2}acsinB$

b) $\frac{S_{ABC}}{S_{AMN}}=\frac{AB.AC}{AM.AN}$

 

P/s: Làm bài phải vẽ hình, bài làm xong được tô màu đỏ


Bài viết đã được chỉnh sửa nội dung bởi Viet Hoang 99: 16-05-2014 - 19:58


#3
einstein627

einstein627

    Trung sĩ

  • Thành viên
  • 102 Bài viết

Bài tập:

$1/$ Cho hình thang $ABCD$ có $BC//AD$; $BC<AD$, kéo dài $AC$ về phía $C$ và lấy $P$ tùy ý. Gọi $K;L$ là trung điểm $BC;AD$. $PK\cap AB\equiv M$; $PL\cap CD\equiv N$

Cmr: $MN//BC$

 

 

Áp dụng định lý Menelauyt cho 2 tam giác CAD và tam giác ABC (có 2 cát tuyến PKM và PNL) ta có

$\frac{NC}{ND}\frac{LD}{LA}\frac{AP}{PC}=1$ (1)

$\frac{MB}{MA}\frac{AP}{PC}\frac{KC}{BK}=1$  (2)

K là trung điểm BC

L là trung điểm AD

$\frac{BK}{KC}=\frac{AL}{LD}=1$

Nên từ 1 và 2 ta có

$\frac{NC}{ND}\frac{AP}{PC}=1=\frac{MB}{MA}\frac{AP}{PC}\Rightarrow \frac{NC}{ND}=\frac{MB}{MA}$

nên ta có MN song song BC (dpcm)

p/s vẽ hình mệt thế

untitled.PNG


Bài viết đã được chỉnh sửa nội dung bởi Viet Hoang 99: 28-04-2014 - 19:06

-Học từ ngày hôm qua, sống ngày hôm nay, hi vọng cho ngày mai. Điều quan trọng nhất là không ngừng đặt câu hỏi.

-Albert Einstein

 
-Khi Bạn Sắp Bỏ Cuộc, Hãy Nhớ Tới Lý Do Khiến Bạn Bắt Đầu.

 


#4
einstein627

einstein627

    Trung sĩ

  • Thành viên
  • 102 Bài viết

 

Bài tập:

$4/$ Cho tam giác $ABC$ cạnh $a;b;c$. $M\in AB; N\in AC$

Cmr:

a) $S_{ABC}=\frac{1}{2}bcsinA=\frac{1}{2}absinC=\frac{1}{2}acsinB$

b) $\frac{S_{ABC}}{S_{AMN}}=\frac{AB.AC}{AM.AN}$

a,Kẻ đường cao BH ta có sin$\widehat{A}$=$\frac{BH}{AB}$

Mặt khác ta có S$\Delta$ABC=$\frac{BH.AC}{2}=\frac{HB}{AB}.\frac{1}{2}.AB.AC=\frac{1}{2}Sin\widehat{A}.bc$
tương tự ta có $S\Delta ABC=\frac{1}{2}Sin\widehat{A}.bc=\frac{1}{2}Sin\widehat{B}.ac=\frac{1}{2}Sin\widehat{C}.ab$

b,Áp dụng câu a ta có

$\frac{S\Delta ABC}{S\Delta AMN}=\frac{\frac{1}{2}Sin\widehat{A}.AB.AC}{\frac{1}{2}Sin\widehat{A}.AM.AN}=\frac{AB.AC}{AM.AN}$
(dpcm)

untitled.PNG


Bài viết đã được chỉnh sửa nội dung bởi Viet Hoang 99: 28-04-2014 - 20:26

-Học từ ngày hôm qua, sống ngày hôm nay, hi vọng cho ngày mai. Điều quan trọng nhất là không ngừng đặt câu hỏi.

-Albert Einstein

 
-Khi Bạn Sắp Bỏ Cuộc, Hãy Nhớ Tới Lý Do Khiến Bạn Bắt Đầu.

 


#5
lovemathforever99

lovemathforever99

    Thượng sĩ

  • Thành viên
  • 216 Bài viết

 

$2/$ Cho tam giác $ABC$ có $AB>AC$. Qua $C$ kẻ đường thẳng vuông góc với phân giác $BE$ của $\widehat{ABC}$ tại $F$, cắt trung tuyến $BD$ tại $G$. 

Cmr: $DF$ đi qua trung điểm của $GE$

cm FD qua tđ EG.jpg

Gọi $H$ là giao điểm $CF$ và $AB$.

$\Rightarrow \bigtriangleup BCH$ cân tại $B$.

$\Rightarrow F$ là trung điểm $CH$

$\Rightarrow DF$ là đường trung bình $\bigtriangleup CAH$

$\Rightarrow DF//AB$ 

 

Gọi $I$ là giao điểm $DF$ và $BC$. 

$\Rightarrow I$ là trung điểm $BC$.

 

Áp dụng $Ce-va$ cho tam giác $BDC$ 

$\Rightarrow \frac{DG}{GB}=\frac{DE}{CE}\Rightarrow GE//BC$

Bây giờ Ta-let giữa 2 tam giác $DBI$ và $DCI$ 

$\Rightarrow DF$ qua trung điểm $EG$ 


Bài viết đã được chỉnh sửa nội dung bởi Viet Hoang 99: 29-04-2014 - 05:50

                                                 ''Chúa không chơi trò xúc xắc.''

Albert Einstein


#6
lovemathforever99

lovemathforever99

    Thượng sĩ

  • Thành viên
  • 216 Bài viết

 

$3/$ Cho tam giác $ABC$, trọng tâm $G$. Đường thẳng đi qua $G$ cắt $AB;AC$ tại $M;N$. Xác định vị trí của $M;N$ sao cho: $P=\frac{AB.AC}{AM.AN}$ đạt giá trị lớn nhất

max.jpg

Kẻ $BE,CF//MN\Rightarrow \bigtriangleup BDE=\bigtriangleup CDF\Rightarrow DE=DF$

 

$\frac{AB}{AM}+\frac{AC}{AN}=\frac{AE}{AG}+\frac{AF}{AG}=\frac{AD-DE+AD+DF}{AG}=\frac{2AD}{AG}=3$

 

$\Rightarrow P=\frac{AB.AC}{AM.AN}\leq \frac{1}{4}(\frac{AB}{AM}+\frac{AC}{AN})^{2}=\frac{9}{4}$

 

Dấu = khi $MN//BC$.


                                                 ''Chúa không chơi trò xúc xắc.''

Albert Einstein


#7
Viet Hoang 99

Viet Hoang 99

    $\textbf{Trương Việt Hoàng}$

  • Điều hành viên THPT
  • 2291 Bài viết

$5/$ Cho tam giác cân có góc ở đỉnh bằng $20^o$. Cạnh đáy có độ dài là $a$, cạnh bên có độ dài là $b$.

Cmr: $a^3+b^3=3ab^2$
 

$6/$ Cho tam giác $ABC$, $M$ nằm trong tam giác. Gọi $A_1;B_1;C_1$ là hình chiếu của $M$ trên $BC;CA;AB$.

1.Đặt $MA_1=x;MB_1=y;MC_1=z;BC=a;AC=b;AB=c$

Xác định $M$ để $P=\frac{a}{x}+\frac{b}{y}+\frac{c}{z}$ đạt $Min$

2.Xác định $M$ để biểu thức $Q=AC_1^2+BA_1^2+CB_1^2$ đạt $Min$

3.ĐƯờng thẳng $AM;BM;CM$ cắt các cạnh tam giác $ABC$ lần lượt tại $D;E;F$. Tìm $Min$
a) $T=\frac{AM}{MD}+\frac{BM}{ME}+\frac{CM}{MF}$
b) $R=\sqrt{\frac{AM}{MD}}+\sqrt{\frac{BM}{ME}}+\sqrt{\frac{CM}{MF}}$


Bài viết đã được chỉnh sửa nội dung bởi Viet Hoang 99: 29-05-2014 - 12:11


#8
Super Fields

Super Fields

    Thiếu úy

  • Thành viên
  • 526 Bài viết

$5/$ Cho tam giác cân có góc ở đỉnh bằng $20^o$. Cạnh đáy có độ dài là $a$, cạnh bên có độ dài là $b$.

Cmr: $a^3+b^3=3ab^2$

$\boxed 5$

 

1.png

 

Dựng $\angle CBx=20^o$

 

Hạ $AH$ vuông góc $Bx$ tại $H$

 

$D$ là giao điểm của $Bx$ và $AC$

 

Ta có:

 

$\frac{BH}{AB}=cos(60^o)=\frac{1}{2} \Rightarrow BH=\frac{b}{2}$

 

Dễ chứng minh $\Delta BDC$ cân 

 

$\Rightarrow BD=BC=a$

 

$\Rightarrow DH=BH-DH=\frac{b}{2}-a=\frac{b-2a}{2}$

 

Áp dụng Pythagore vào $\Delta ABH$ tính được $AH^2=\frac{3b^2}{4}$

 

Vậy $AD^2=b^2-ab+a^2$                                                        $(1)$

 

Mà $\Delta ABC \sim \Delta BCD$

 

$\Rightarrow \frac{DC}{BC}=\frac{BD}{AC}$

 

$\Rightarrow CD=\frac{a^2}{b}$

 

$\Rightarrow AD=b-\frac{a^2}{b}$

 

$\Rightarrow AD^2= b^2 - 2b\frac{a^2}{b}+ \frac{a^4}{b^2}$                          $(2)$

 

Từ $(1)$ = $(2)$ có $3a^2-ab-\frac{a^4}{b^2}=0$ 

 

Nhân 2 vế với $b^2$ rồi chia cho $a$ ta có đpcm.

 

 

Viet Hoang 99:
Làm xong bài thì check lại nhé, toàn viết nhầm, còn tắt chưa viết tỉ số đồng dạng nữa :P

Bước cuối phải nói rõ nhân $b^2$ chia $a$ nữa.

P/s: Không cần viết cách 1 dòng như vậy, tốn diện tích :)


Bài viết đã được chỉnh sửa nội dung bởi Viet Hoang 99: 29-04-2014 - 20:07

$\dagger$God made the integers, and else is the work of man.$\dagger$


$\boxed{\textrm{My Blog}}$


#9
Super Fields

Super Fields

    Thiếu úy

  • Thành viên
  • 526 Bài viết

$6/$ Cho tam giác $ABC$, $M$ nằm trong tam giác. Gọi $A_1;B_1;C_1$ là hình chiếu của $M$ trên $BC;CA;AB$.

1.Đặt $MA_1=x;MB_1=y;MC_1=z;BC=a;AC=b;AB=c$

Xác định $M$ để $P=\frac{a}{x}+\frac{b}{y}+\frac{c}{z}$ đạt $Min$

$\boxed 6_{1}$

 

$P=\frac{a}{x}+\frac{b}{y}+\frac{c}{z}=\frac{a^2}{ax}+\frac{b^2}{by}+\frac{c^2}{cz} \geq \frac{(a+b+c)^2}{2S_{ABC}}$

 

Dấu đẳng thức xảy ra $\Leftrightarrow \frac{1}{x}=\frac{1}{y}=\frac{1}{z} \Leftrightarrow x=y=z \Leftrightarrow M$ là tâm đ/tròn nội.

 

Viet Hoang 99:
Tâm đường tròn nội chứ sao lại ngoại @@

Vẫn như trên, làm xong bài thì check lại


Bài viết đã được chỉnh sửa nội dung bởi Viet Hoang 99: 29-04-2014 - 20:09

$\dagger$God made the integers, and else is the work of man.$\dagger$


$\boxed{\textrm{My Blog}}$


#10
Super Fields

Super Fields

    Thiếu úy

  • Thành viên
  • 526 Bài viết

3.Đường thẳng $AM;BM;CM$ cắt các cạnh tam giác $ABC$ lần lượt tại $D;E;F$. Tìm $Min$

a) $T=\frac{AM}{MD}+\frac{BM}{ME}+\frac{CM}{MF}$

 

$\boxed{6_{3}}a$

 

$T=\frac{S_{ABM}}{S_{BMD}}+\frac{S_{BMC}}{S_{CME}}+\frac{S_{AMC}}{S_{AMF}}=\frac{S_{1}^2}{S_{4}.S_{1}}+\frac{S_{2}^2}{S_{5}.S_{2}}+\frac{S_{3}^2}{S_{6}.S_{3}}\geq \frac{S^2}{S_{4}.S_{1}+S_{5}.S_{2}+S_{6}.S_{3}}\geq \frac{2S^2}{S+S_{1}+S_{2}+S_{3}}\geq \frac{4S}{3}$

 

$\Leftrightarrow M$ là tâm $\Delta ABC$ đều

 

Viet Hoang 99:

 

Lại còn không biết $S_1;...S_6$ là gì nữa @@

Đặt $S_1=S_{ABM};S_{2}=S_{BMC};S_{3}=S_{AMC};S_{4}=S_{BMD};S_{5}=S_{CME};S_{6}=S_{AMF}$

Mà sai rồi, xem bài sửa bên dưới


Bài viết đã được chỉnh sửa nội dung bởi Viet Hoang 99: 29-04-2014 - 20:59

$\dagger$God made the integers, and else is the work of man.$\dagger$


$\boxed{\textrm{My Blog}}$


#11
Viet Hoang 99

Viet Hoang 99

    $\textbf{Trương Việt Hoàng}$

  • Điều hành viên THPT
  • 2291 Bài viết

$6/$ Cho tam giác $ABC$, $M$ nằm trong tam giác. Gọi $A_1;B_1;C_1$ là hình chiếu của $M$ trên $BC;CA;AB$.

1.Đặt $MA_1=x;MB_1=y;MC_1=z;BC=a;AC=b;AB=c$

Xác định $M$ để $P=\frac{a}{x}+\frac{b}{y}+\frac{c}{z}$ đạt $Min$

2.Xác định $M$ để biểu thức $Q=AC_1^2+BA_1^2+CB_1^2$ đạt $Min$

3.ĐƯờng thẳng $AM;BM;CM$ cắt các cạnh tam giác $ABC$ lần lượt tại $D;E;F$. Tìm $Min$
a) $T=\frac{AM}{MD}+\frac{BM}{ME}+\frac{CM}{MF}$
b) $R=\sqrt{\frac{AM}{MD}}+\sqrt{\frac{BM}{ME}}+\sqrt{\frac{CM}{MF}}$

Bài nào cũng phải vẽ hình nhé

$6/$

h.PNG

1.

$\boxed 6_{1}$

 

$P=\frac{a}{x}+\frac{b}{y}+\frac{c}{z}=\frac{a^2}{ax}+\frac{b^2}{by}+\frac{c^2}{cz} \geq \frac{(a+b+c)^2}{2S_{ABC}}$

 

Dấu đẳng thức xảy ra $\Leftrightarrow \frac{1}{x}=\frac{1}{y}=\frac{1}{z} \Leftrightarrow x=y=z \Leftrightarrow M$ là tâm đ/tròn nội.

 

Viet Hoang 99:
Tâm đường tròn nội chứ sao lại ngoại @@

Vẫn như trên, làm xong bài thì check lại

2.

$AC_1^2=AM^2-MC_1^2-AB_1^2+MB_1^2-MC_1^2$
Tương tự: ...

$\Rightarrow Q=AC_1^2+BA_1^2+CB_1^2=...=AB_1^2+BC_1^2+CA_1^2$

$2Q=Q+Q=AC_1^2+BA_1^2+CB_1^2+AB_1^2+BC_1^2+CA_1^2=(AC_1^2+BC_1^2)+(BA_1^2+CA_1^2)+(AB_1^2+CB_1^2)\geq \frac{AB^2}{2}+\frac{BC^2}{2}+\frac{CA^2}{2}\Rightarrow Q\geq \frac{a^2+b^2+c^2}{4}$

(Áp dụng BĐT $m^2+n^2\geq \frac{(m+n)^2}{4}$)

Dấu = có khi:$ AC_1=BC_1$; $BA_1=CA_1$; $AB_1=CB_1$

$\Rightarrow $ $M$ là tâm ngoại tam giác $ABC$.

 

3.

 

a)

 

 

$\boxed{6_{3}}a$

 

$T=\frac{S_{ABM}}{S_{BMD}}+\frac{S_{BMC}}{S_{CME}}+\frac{S_{AMC}}{S_{AMF}}=\frac{S_{1}^2}{S_{4}.S_{1}}+\frac{S_{2}^2}{S_{5}.S_{2}}+\frac{S_{3}^2}{S_{6}.S_{3}}\geq \frac{S^2}{S_{4}.S_{1}+S_{5}.S_{2}+S_{6}.S_{3}}\geq$
$\frac{2S^2}{S+S_{1}+S_{2}+S_{3}}$  chỗ này là $\frac{2S^2}{S+S_{4}+S_{5}+S_{6}}$
$\geq \frac{4S}{3}$ Là sao

 

$\Leftrightarrow M$ là tâm $\Delta ABC$ đều

 

 

Viet Hoang 99

Lại còn không biết $S_1;...S_6$ là gì nữa @@

Đặt $S_1=S_{ABM};S_{2}=S_{BMC};S_{3}=S_{AMC};S_{4}=S_{BMD};S_{5}=S_{CME};S_{6}=S_{AMF}$

Sai rồi, 

 

Làm như sau:
$\frac{AM}{MD}=\frac{S_{ABM}}{S_{BMD}}=\frac{S_{ACM}}{S_{DMC}}=\frac{S_1+S_3}{S_2}$

$\Rightarrow T=\frac{AM}{MD}+\frac{BM}{ME}+\frac{CM}{MF}=\sum (\frac{S_1}{S_2}+\frac{S_2}{S_1})\geq 2+2+2=6$
 
Dấu = có khi: $M$ là trọng tâm tam giác $ABC$
 
 

b)

Đặt $S_{ABM}=a^2;S_{ACM}=b^2;S_{BCM}=c^2$
$\Rightarrow R=\sum \sqrt{\frac{a^2+b^2}{c^2}}\geq \sum \sqrt{\frac{(a+b)^2}{2c^2}}=\sum \frac{a+b}{c\sqrt{2}}=\frac{1}{\sqrt{2}}\sum (\frac{a}{b}+\frac{b}{a})\geq \frac{6}{\sqrt{2}}$

 

Dấu = có khi: $M$ là trọng tâm tam giác $ABC$

P/s: Hình vẽ đơn giản thì vẽ nhỏ thôi.


Bài viết đã được chỉnh sửa nội dung bởi Viet Hoang 99: 29-04-2014 - 20:57


#12
Viet Hoang 99

Viet Hoang 99

    $\textbf{Trương Việt Hoàng}$

  • Điều hành viên THPT
  • 2291 Bài viết

$7/$ Cho tam giác $ABC$. $D;E;F$ lần lượt thuộc cạnh $BC;CA;AB$. $AD\cap CF\equiv M;AD\cap BE\equiv N;BE\cap CF\equiv P$

Biết $S_{MNP}=S_{AMF}=S_{BDN}=S_{CPE}=1$
Tính $S_{ABC}$

 

$8/$ Cho lục giác $ABCDEF$ có các cạnh đối song song. Gọi $H;K$ là hình chiếu của $A;B$ trên $CE;DF$. Cm: $\frac{DF}{CE}=\frac{AH}{BK}$


Bài viết đã được chỉnh sửa nội dung bởi Viet Hoang 99: 30-05-2014 - 08:27


#13
Trang Luong

Trang Luong

    Đại úy

  • Thành viên
  • 1834 Bài viết

$9/$ Cho $\Delta ABC$, $D,E$ di động trên tia $BA,CA$ sao cho $BD=EC$.

a) Vẽ hình bình hành $BDEM$. Tim tập hợp điểm $M$

b) Tìm vị trí $D,E$ để $DE$ min


Bài viết đã được chỉnh sửa nội dung bởi Viet Hoang 99: 05-07-2014 - 20:48

"Nếu bạn hỏi một người giỏi trượt băng làm sao để thành công, anh ta sẽ nói với bạn: ngã, đứng dậy là thành công"
Issac Newton

#14
Phuong Mark

Phuong Mark

    Thượng sĩ

  • Thành viên
  • 225 Bài viết

10) Cho $(O,R)$ và đường thẳng $(d)$ cắt đường tròn tai $2$ điểm $A$ và $B$.Từ điểm $M$ nắm trên đường thẳng $d$ và ở ngoài $(O,R)$ kẻ hai tiếp tuyến $MP$ và $MQ$ đến đường tròn , trong đó $P$ và $Q$ là các tiếp điểm.

1, Gọi $I$ là giao điểm của $MO$ với $(O,R)$ . Chứng minh $I$ là tâm của đường tròn nội tiếp $\Delta MPQ$.

2, Xác định vị trí điểm $M$ trên đường thẳng $d$ để tứ giác $MPOQ$ là hình vuông.

3, Chứng minh rằng khi điểm $M$ di chuyển trê đường thẳng $d$ thì tâm đường tròn ngoại tiếp $\Delta MPQ$ chạy trên một đường thẳng cố định. 

 

 

11) Trên mỗi nửa đường tròn đường kính $AB$ của đường tròn (O) lấy một điểm tương ứng là $C4 và $D$ thỏa mãn.

                                            $AC^{2}+BD^{2}=AD^{2}+BC^{2}$

Gọi $K$ là trung điểm của $BC$. Hãy xác định vị trí các điểm $C$ và $D$ trên đường tròn $(O)$ để đường thẳng $DK$ đi qua trung điểm của $AB$.

 

 

12) Cho đường tròn $(O)$ đường kính $AB=2R$. Một điểm $M$ chuyển động trên đường tròn $(O)$ ( M khác A và B) . Gọi $H$ là hình chiếu vuống góc cảu $M$ trên đường kính $AB$.Vẽ đường tròn có tâm là $(M)$ và bán kính là $MH$.Từ $A$ và $B$ lần lượtker các tiếp tuyến $AD$ và $BC$ đến đường tròn (D và C là các tiếp điểm ).

1, Chứng minh rằng khi $M$ di chuyển trên đường tròn $(O)$ thì $AD+BC$ có giá trị không đổi.

2, Chứng minh rằng $CD$ là tiếp tuyến của đường tròn $(O)$.

3, Chứng minh với bất kì vị trí nào của $M$ trên đường tròn (O) luôn có bất đẳng thức $AD.BC \leq R^{2}$. Xác định vị trí của $M$ trên đường tròn $(O)$ để đẳng thức xảy ra.

4, Trên đường tròn $(O)$ lấy điểm $N$ cố định. Gọi $I$ là trung điểm của $MN$ và $P$ là hình chiếu vuông góc của $I$ trên $MB$ . Khi $M$ di chuyển trên $(O)$ thì $P$ chạy trên đường tròn nào?

 

 

13) Cho nửa đường tròn đường kính $AB$. Gọi $P$ là điểm chính giữa của cung $AB$, $M$ là điểm chuyển động trên cung $BP$. Trên đoạn $AM$ lấy điểm $N$ sao cho $AN=BM$.

1, Chứng minh tỉ số $\frac{NP}{MN}$ có giá trị không đổi khi điểm $M$ di chuyển trên cung $BP$. Tìm giá trị không đổi ấy?

2, Tìm tập hợ các điểm $N$ khi $M$ di chuyển trên cung $BP$ 

 

 

14) Cho tứ giác $ABCD$ có $AB=AD$ và $CB=CD$

1, Chứng minh rằng:

a, Tứ giác $ABCD$ ngoại tiếp được một đường tròn

b, Tứ giác $ABCD$ nội tiếp được trong một đường tròn khi và chỉ khi $AB$ và $BC$ vuông góc với nhau.

2, Giả sử $AB \perp BC$ . Gọi $(N)$ là đuờng tròn nội tiếp và $(M,R)$ là đường tròn khi tiếp tứ giác $ABCD$. Chứng minh:

a, $AB+BC=r+ \sqrt{r^{2}+4R^{2}}$

b, $MN^{2}=R^{2}+r^{2} \sqrt{r^{2}+4R^{2}}$

 

 

15) Gọi $A,B,C$ là góc của tam giác $ABC$.

Tìm điểu kiện của tam giác $ABC$ để biểu thức :

$P=sin \frac{A}{2} sin \frac{B}{2} sin \frac{C}{2}$

đạt giá trị lớn nhất . Tìm giá trị lớn nhất ấy?

 

 

16) CHo hình vuông $ABCD$ 

1, Với mỗi điểm $M$ cho trước trên cạnh $AB$( khác $A$ và $B$). Trên cạnh $AD$ lấy điểm $N$ sao cho cho vi tam giác $AMN$ gấp 2 lần chu vi hình vuông đã cho.

2, Kẻ đường thẳng sao cho mỗi đường thẳng này chia hình vuông đã cho thành $2$ tứ giác có tỉ số diện tích bằng $\frac{2}{3}$. Chứng minh rằng trong $9$ đường thẳng có ít nhất $3$ đường đồng quy.

 

 

17) Cho tam giác nội tiếp trong $(O;R)$ với $BC=a,AC=b,Ab=c$. lấy điểm $I$ bất kì ở phía trong tam giác và gọi $x,y,z$ lần lượt là khoảng cách từ $I$ đến $BC,AC,AB$.

chứng minh: $\sqrt {x}+\sqrt{y}+\sqrt{z} \leq \sqrt{ \frac{a^{2}+b^{2}+c^{2}}{2R}}$

 

 

18) Cho $(O;R)$ và 2 điểm A,B nằm ở ngoài $(O)$ với $OA=OB=2R$ và $OA$ vuông góc với $OB$ . Xác định vị trí của $M$ thuộc $(O)$ sao cho biểu thức : $P=MA+2MB$ đạt giá trị nhỏ nhất. Tìm giá trị nhỏ nhất ấy?

 

 

19) Cho tam giác $ABC$ nhọn . Gọi $D$ là trung điểm cạnh $BC$,$M$ là điểm tùy ý trênn cạnh $AB$ (không trùng với các đỉnh $A$ và $B$) . Gọi $H$ là giao điểm của các đoạn thẳng $AD$ và $CM$. Chứng minh rằng nếu tứ giá $MBHD$ nội tiếp được trong một đường tròn thì có bất đẳng thức $BC< \sqrt{2} AC$


Bài viết đã được chỉnh sửa nội dung bởi Viet Hoang 99: 09-07-2014 - 07:20

Hẹn ngày tái ngộ VMF thân yêu !

 

 

 


#15
HungNT

HungNT

    Thượng sĩ

  • Thành viên
  • 273 Bài viết

10) Cho $(O,R)$ và đường thẳng $(d)$ cắt đường tròn tai $2$ điểm $A$ và $B$.Từ điểm $M$ nắm trên đường thẳng $d$ và ở ngoài $(O,R)$ kẻ hai tiếp tuyến $MP$ và $MQ$ đến đường tròn , trong đó $P$ và $Q$ là các tiếp điểm.

1, Gọi $I$ là giao điểm của $MO$ với $(O,R)$ . Chứng minh $I$ là tâm của đường tròn nội tiếp $\Delta MPQ$.

2, Xác định vị trí điểm $M$ trên đường thẳng $d$ để tứ giác $MPOQ$ là hình vuông.

3, Chứng minh rằng khi điểm $M$ di chuyển trê đường thẳng $d$ thì tâm đường tròn ngoại tiếp $\Delta MPQ$ chạy trên một đường thẳng cố định. 

 

 

$10/$

$a/$ Ta có $OI\bot PQ$ nên $I$ là điểm chính giữa cung $PQ\Rightarrow $ cung $PI$ = cung $IQ$

=> $\widehat{IPM}=\widehat{IPQ}, \widehat{IQM}=\widehat{IQP}$ do chắn 2 cung bằng nhau

=> IP và IQ là các tia phân giác => I là tâm dường tròn nội tiếp $\Delta MPQ$

b / OPMQ là hình vuông <=> $\widehat{POQ}=90^{\circ}$ =>$\widehat{POM}=45^{\circ}\Rightarrow OM=\frac{OP}{cosPOM}=\frac{R}{cos45^{\circ}}=R\sqrt{2}$

c / Gọi tâm đường tròn ngoại tiêp $\Delta MPQ$ là H, vì OPMQ nội tiếp nên H là tâm đường tròn

=> H trung điểm OM. Gọi N là trung điểm OA=> NH//BC [đường tb] => NH cố định=> H luôn di chuyển trên 1 đường thẳng cố định

 

Kẹp $ vào các điểm cho đẹp, như mình đã sửa

Bài làm phải có hình vẽ, trích dẫn, ghi STT bài, mình sửa tất cả trừ hình thì chưa đăng, bạn tự đăng hình lên đi, đã có hướng dẫn ở #1


Bài viết đã được chỉnh sửa nội dung bởi Viet Hoang 99: 16-05-2014 - 20:07


#16
nguyenhongsonk612

nguyenhongsonk612

    Thượng úy

  • Thành viên
  • 1451 Bài viết

Bài 20:

Cho $2$ đường tròn cố định $(O;R)$ và $(I;R')$ cắt nhau tại 2 điểm A và B, biết (I) đi qua O. Vẽ 2 đường kính AE và BF của (O); Gọi C là điểm di động trên cung EF của (O), biết $A \notin$ cung EF, $C\neq {E;F}$. Đường thẳng CO cắt (O) và (I) thứ tự tại K và D. $(K\neq C;D\neq O)$

a) Chứng minh rằng $\widehat{CAD}+\widehat{OBK}=180^o$

b) Chứng minh K là tâm đường tròn nội tiếp $\bigtriangleup ABD$

c) Xác định vị trí điểm C trên cung EF sao cho diện tích tứ giác ACBD lớn nhất.

P/s: Bài hay!

 

 

 

_____________

Chú ý: Onl cũng khá thường xuyên :P , bạn 

HungNT

nói đề sai mà mãi không sửa à ?

Mình đã sửa


Bài viết đã được chỉnh sửa nội dung bởi Viet Hoang 99: 16-05-2014 - 20:14

"...Từ ngay ngày hôm nay tôi sẽ chăm chỉ học hành như Stardi, với đôi tay nắm chặt và hàm răng nghiến lại đầy quyết tâm. Tôi sẽ nỗ lực với toàn bộ trái tim và sức mạnh để hạ gục cơn buồn ngủ vào mỗi tối và thức dậy sớm vào mỗi sáng. Tôi sẽ vắt óc ra mà học và không nhân nhượng với sự lười biếng. Tôi có thể học đến phát bệnh miễn là thoát khỏi cuộc sống nhàm chán khiến mọi người và cả chính tôi mệt mỏi như thế này. Dũng cảm lên! Hãy bắt tay vào công việc với tất cả trái tim và khối óc. Làm việc để lấy lại niềm vui, lấy lại nụ cười trên môi thầy giáo và cái hôn chúc phúc của bố tôi. " (Trích "Những tấm lòng cao cả")

~O) 


#17
HungNT

HungNT

    Thượng sĩ

  • Thành viên
  • 273 Bài viết

Bài 20:

Cho $2$ đường tròn cố định $(O;R)$ và $(I;R')$ cắt nhau tại 2 điểm A và B, biết (I) đi qua O. Vẽ 2 đường kính AE và BF của (O); Gọi C là điểm di động trên cung EF của (O), biết $A \notin$ cung EF, $C\neq {E;F}$. Đường thẳng CO cắt (O) và (I) thứ tự tại K và D. $(K\neq C;D\neq O)$

a) Chứng minh rằng $\widehat{CAD}+\widehat{OBK}=180^o$

b) Chứng minh K là tâm đường tròn nội tiếp $\bigtriangleup ABD$

c) Xác định vị trí điểm C trên cung EF sao cho diện tích tứ giác ACBD lớn nhất.

P/s: Bài hay!

 

20)

h.PNG

a / Ta có $\widehat{ABO}=\widehat{ADC}, \widehat{ABK}=\widehat{ACD}$ do cùng chắn cung AK, AO

=>$\widehat{CAD}+\widehat{ABO}+\widehat{ABK}=\widehat{CAD}+\widehat{ACD}+\widehat{ADC}$$= 180^{\circ}$

hay $\widehat{CAD}+\widehat{OBK}=180^{\circ}$

b / Ta có OI là trung trực của AB nên O là điểm chính giữa cung AB của (I)

=>DK là phân giác của $\widehat{ADB }$ [1]

Kẻ tia Ax là tia đối của AD, ta có $\widehat{CAx}=\widehat{ACD}+\widehat{ADC}=\widehat{OBK}$

mà $\widehat{OBK}= \widehat{OKB}=\widehat{CAB}=> \widehat{CAx}=\widehat{CAB}$

=> BK là phân giác góc ngoài tam giác ABD

mà AK$\bot$AC nên AK là phân giác của $\widehat{BAD}$ [2]

Từ [1], [2] => K là tâm đường tròn nội tiếp tam giác ABD [ko phải ABC nhá]

c / Hạ BH, AM vuông góc CD

S ACBD = S ACD + S BCD = $\frac{1}{2}\left ( BH+AM \right )CD\leq AB.$(CO+2OI) 

[ vì $BH+AM\leq AB, CD=CO+OD\leq CO+2OI$ ] 

Dấu "=" xảy ra khi C,O,I,D thảng hàng hay C thuộc đường thẳng OI

 

Chú ý: Làm xong bài thì nhớ đọc lại nhé, toàn viết nhầm thôi!

Bài làm phải có hình vẽ, cách vẽ đã có ở #1

Làm bài nhớ ghi STT và trích dẫn

Mình đã bổ sung tất cả cho bài bạn rồi!


Bài viết đã được chỉnh sửa nội dung bởi Viet Hoang 99: 16-05-2014 - 19:56


#18
Phuong Mark

Phuong Mark

    Thượng sĩ

  • Thành viên
  • 225 Bài viết

19) Cho tam giác $ABC$ nhọn . Gọi $D$ là trung điểm cạnh $BC$,$M$ là điểm tùy ý trênn cạnh $AB$ (không trùng với các đỉnh $A$ và $B$) . Gọi $H$ là giao điểm của các đoạn thẳng $AD$ và $CM$. Chứng minh rằng nếu tứ giá $MBHD$ nội tiếp được trong một đường tròn thì có bất đẳng thức $BC< \sqrt{2} AC$

                                                                                     Bài làm

h.jpg

$BD=DC=x$. Nếu tứ giác $BMHD$ nội tiếp được một đường tròn ta phải đi chứng minh $ BC^{2} \leq 2AC^{2}\Rightarrow 4x^{2} \leq AC^{2}\Rightarrow 2x^{2} \leq 2AC^{2}$ $(1)$

mà ta lại có: $2x^{2}=CH.CM$
Vậy chứng CM: $CH.CM \leq AC^{2}$(2) kẻ $Ax \bot AC$ cắt $CM$ ở $M'$ . Kẻ $AK \bot CM$ tại $K$.
Xét tam giác vuông $AM'C$ ta có: $AC^{2}=CM'.CK$
Vì góc $MAC$ nhọn và góc $M'AC=90^{*}$ $\Rightarrow M$ nằm giữa $M'$ và $C$ $\Rightarrow CM \leq CM'$ . Mà ta chứng minh được: góc $AHC$ là góc tù $\Rightarrow M$ nằm giữa $K$ và $C\Rightarrow $ $CH \leq CK$
Vì $AC^{2}= CM'.CK\Rightarrow$ $CM.CH \leq AC^{2}$ ( vậy $(2)$ được $CM$) $\Leftrightarrow $ $(1)$ được chứng minh
Vậy: $BC^{2} \leq 2AC^{2}\Rightarrow BC \leq \sqrt{2}AC$

 

:icon6: Mình mới nghĩ ra nên làm luôn kẻo quên

 

------------------

Viet Hoang 99:
Chú ý: Đăng hình như ở #1, dùng URL ảnh bị hỏng sau một thời gian.

 


Bài viết đã được chỉnh sửa nội dung bởi Viet Hoang 99: 29-05-2014 - 20:21

Hẹn ngày tái ngộ VMF thân yêu !

 

 

 


#19
HungNT

HungNT

    Thượng sĩ

  • Thành viên
  • 273 Bài viết

11/ Ta có $AC^{2}=AB^{2}-BC^{2}$

$AD^{2}=AB^{2}-BD^{2}$

mà $AC^{2}+BD^{2}=AD^{2}+BC^{2}$

$\Rightarrow AB^{2}-BC^{2}+BD^{2}=AB^{2}-BD^{2}+BC^{2}$

$\Rightarrow 2BD^{2}=2BC^{2}\Rightarrow BD=BC$

$\Rightarrow \Delta BCD$ cân tại $B$

lại có $DK$ trung tuyến đi qua $O$ nên đồng thời là trung trực $\Rightarrow \Delta BCD$ đều

Gọi $H$ là giao điểm $AB$ và $CD$ $\Rightarrow OH=\frac{1}{2}OB=OA$

Vậy vị trí $C$,$D$ thỏa mãn đề bài khi $CD$ là trung trực của $OA$

Hình gửi kèm

  • untitled.PNG

Bài viết đã được chỉnh sửa nội dung bởi HungNT: 17-05-2014 - 16:09


#20
HungNT

HungNT

    Thượng sĩ

  • Thành viên
  • 273 Bài viết

13/a/ Xét $\Delta ANP$ và $\Delta BMP$ có

$AN=MB$ và $AB=BP$ 

$\widehat{NAP}=\widehat{MBP}$ (cùng chắn cung $PM$)

$\Rightarrow \Delta ANP=\Delta BMP$ (c-g-c)

$\Rightarrow PN= PM$ và $\widehat{NPA}=\widehat{MPB}$

$\Rightarrow \widehat{MBP}+\widehat{BPN}=\widehat{NPA}+\widehat{BPN}=90^{\circ}$

$\Rightarrow \Delta NPM$ vuông cân tại $P$ $\Rightarrow \frac{NP}{MN}= \frac{1}{\sqrt{2}}$

b/ Vì $\widehat{PNA}=\widehat{PMB}=\frac{sdPB}{2}=135^{\circ}$

mà AP cố định nên tập hợp điểm N là cung chứa góc $135^{\circ}$ dựng trên $AP$

Hình gửi kèm

  • untitled2.PNG

Bài viết đã được chỉnh sửa nội dung bởi HungNT: 17-05-2014 - 16:33





8 người đang xem chủ đề

0 thành viên, 8 khách, 0 thành viên ẩn danh